[Calculus] 6. Exponential and Logarithm

Lemma 6.1 Let \(m\in\N\). For every \(a\in[0,\infty)\), there exists a unique \(c\in[0,\infty)\), such that \(c^m=a\).

Proof (Uniqueness)

Consider a map \(f:[0,\infty)\to [0,\infty)(\sube\R)\), from \(x\) to \(x^m\). It is needed to show the map is injective.

Given two \(x_1\ne x_2\in[0,\infty)\). Then \(\frac{x_1}{x_2}\ne1\), i.e., \(\left(\frac{x_1}{x_2}\right)^m\ne1\), thus \(x_1^m\ne x_2^m\).

(Existence)

Case 1, \(a=0\), then we can find \(c=0\).

Case 2, \(a>0\). Since the map \(f:x\mapsto x^m\) is continuous. (Since \(f(x)=x\) is continuous, the products by itself is also continous.)

\(f(0) = 0 < a\). On the other hand, \(f(a+1)=(a+1)^m > a\).

By the intermediate value theorem, \(\exists c\in(0,a+1)[f(c)=c^m=a]\).

Definition 6.2 For any \(c\ge0\), we define \(a^{\frac{1}{m}}\) to be such a number \(c\). In other words, \(c=a^{\frac{1}{m}} \Lrarr c^m=a\land c\ge0\).

More generally, for any \(r\in\Q\), we define \(a^r\) to be \((a^n)^m\) if \(r=\frac{n}{m}\) for some \(n\in\Z,m\in\N\).

Proposition 6.3 Let \(a>0\) and \(r_1,r_2\in\Q\).

\(a^{r_1}a^{r_2}=a^{r_1+r_2}\), \((a^{r_1})^{r_2}=a^{r_1r_2}\). \(r_1 < r_2, a > 1 \Rarr a^{r_1} < a^{r_2}\). If \(a<1\), then \(a^{r_1}>a^{r_2}\). \(\lim_{n\to\infty}a^{\frac{1}{n}}=1\).

Proof Let \(r_1=n_1/m_1, r_2=n_2/m_2\), where \(n_1,n_2\in\Z, m_1,m_2\in\N\).

(1.1) By def, there exist two numbers \(b_1,b_2\), such that \(b_1^{m_1}=a^{n_1},b_2^{m_2}=a^{n_2}\).

Therefore,

\[ (b_1b_2)^{m_1m_2}= (b_1^{m_1})^{m_2}(b_2^{m_2})^{m_1}= a^{n_1m_2}a^{n_2m_1}= a^{n_1m_2+n_2m_1} \]

i.e.,

\[ b_1b_2=\left(a^{n_1m_2+n_2m_1}\right)^{\frac{1}{m_1m_2}} = a^{\frac{n_1}{m_1}+\frac{n_2}{m_2}} = a^{r_1+r_2} \]

And \(b_1b_2\) is actually \(a^{r_1}a^{r_2}\).

(1.2) By def, there exist two number \(b_1,b_2\), such that \(b_1^{m_1}=a^{n_1}\) and \(b_2^{m_2}=b_1^{n_2}\).

Therefore,

\[ b_2^{m_1m_2}= (b_2^{m_2})^{m_1}= b_1^{n_2m_1}= a^{n_1n_2} \]

i.e., \(b_2=a^{r_1r_2}\).

And, \(b_2=b_1^{r_2}=(a^{r_1})^{r_2}\), then the proof is done.

(2)

\[ \frac{a^{r_1}}{a^{r_2}}= a^{r_1-r_2}= \left(\frac{1}{a}\right)^{r_2-r_1} \]

since \(r_1< r _2\), then \(r_2-r_1 > 0\). And

\[ \left(\frac{1}{a}\right)^{r_2-r_1} < 1^{r_2-r_1}=1 \]

Thus, \(a^{r_1} < a^{r_2}\).

(3) Suppose \(a>1\), then \(a^{\frac{1}{n}}>1\).

Let \(a=1+e\) for some positive \(e\in\R\).

Then \(a=(1+e)^n > C_n^0y^0 + C_n^1y^1=1+ny\), by binomial theorem.

Therefore, \(0 < y < \frac{a-1}{n}\).

And \(\lim_{n\to\infty}\frac{a-1}{n}=0\), by Squeeze Test, \(\lim_{n\to\infty}y=0\).

Thus, \(\lim_{n\to\infty}a^{\frac{1}{n}}=1\).

Similarly when \(a<1\).

Definition 6.4 Choose a sequence \(r_n(n\in\N)\), such that \(r_n\in\Q\) for all \(n\), and \(\lim_{n\to\infty}r_n=x\), and then define \(a^x:=\lim_{n\to\infty}a^{r_n}\) if the limit exists.

However, we need to show the above definition is well-defined. There are some probloms needed to be fixed:

Does \(a^{r_n}(n\in\N)\) converge? The result is relevant to the sequence we choose?

Proof

(1) The goal also means showing \(a^{r_n}\) is a Cauchy sequence.

Consider this

\[ \vert a^{r_n}-a^{r_m} | = a^{r_m}|a^{r_n-r_m}-1| \]

Since \(r_n\) is convergent, then it is bounded, i.e., \(\exists M_1,M_2\in\Z\forall n\in\N[M_1 \le r_n \le M_2]\).

Thus, \(a^{r_m}\le\max\{a^{M_1},a^{M_2}\}=: M(>0)\) for all \(n\in\N\).

And we have \(\lim_{n\to\infty}a^{\frac{1}{n}}=1\), i.e., \(\forall\epsilon>0\exists N\in\N[k\ge N \Rarr \left|a^{\frac{1}{k}}-1\right|<\epsilon]\).

Choose the restriction to be \(\frac{\epsilon}{M}\).

Since \(\{r_n\}\) converges, it is a Cauchy sequence, i.e., when \(n,m\) reach big enough, then \(r_n-r_m\) will be small enough, so that \(r_n-r_m<\frac{1}{k}\).

Then we have \(a^{r_n-r_m} < a^{\frac{1}{k}} < 1+\frac{\epsilon}{M}\). Similarly, we can obtain \(1-\frac{\epsilon}{M} < a^{\frac{1}{k}} < a^{r_n-r_m}\). (by Archimedean property?)

Therefore, we have:

\[ \vert a^{r_n}-a^{r_m}| = a^{r_m}|a^{r_n-r_m}-1| < M\cdot\frac{\epsilon}{M}=\epsilon \]

In other words, \(\{a^{r_n}\}\) is Cauchy, then convergent.

(2) Goal: If there is another sequence \(r_n'\) is also convergent to \(x\), then \(\lim_{n\to\infty}a^{r_n'}=\lim_{n\to\infty}a^{r_n}\).

\[ \lim{n\to\infty}a^{r_n-r_n'}=1 \]

Proposition 6.5

Given \(a>0\),

  1. For \(x_1,x_2\in\R\), then \(a^{x_1}a^{x_2}=a^{x_1+x_2}\), and \((a^{x_1})^{x_2}=a^{x_1x_2}\).
  2. If \(x_1 < x_2\), then \(a^{x_1} < a^{x_2}\) when \(a>1\), vice versa for \(a < 1\).
  3. \(f:x\mapsto a^x\) is a continuous map.
  4. \(\lim_{x\to0}a^x=1\).

Proof

Suppose there are two sequences \(b_n,c_n(\in\Q)\), such that \(\lim_{n\to\infty}b_n=x_1\) and \(\lim_{n\to\infty}c_n=x_2\) where \(x_1,x_2\in\R\).

(1.1) \(\lim_{n\to\infty}(a_n+b_n)=x_1+x_2\).

Since \(\lim_{n\to\infty}a^{b_n}=a^{x_1},\lim_{n\to\infty}a^{c_n}=a^{x_2}\), we have

\[ a^{x_1}a^{x_2}=\lim_{n\to\infty}(a^{b_n}a^{c_n})= \lim_{n\to\infty}a^{b_n+c_n}=a^{x_1+x_2} \]

(1.2) Choose \(x_n,y_n\in\Q\), specially \(x_n\) grows non-decreasingly to \(x\) as \(n\to\infty\), so as \(y_n\).

Without loss of generality, supppose \(a>1, x,y>0\).

\(a^x \ge a^{x_n} > 1\), then \((a^x)^y \ge (a^{x_n})^{y_n}\).

By the law of exponentials of rationals, \((a^{x_n})^{y_n} = a^{x_ny_n}\) which converges to \(a^{xy}\) non-decreasingly.

Therefore, \((a^x)^y \ge a^{xy}\), too.

Choose another \(x_y,y_n\), which grows non-increasingly to \(x,y\). After the same operations, we obtain \((a^x)^y \le a^{xy}\).

Thus, \((a^x)^y = a^{xy}\).

(2)

\[ \frac{a^{x_1}}{a^{x_2}}=a^{x_1-x_2} \]

Since \(x_1 - x_2 < 0\) and \(a>1\), \(a^{x_1-x_2}<1\). Vice versa.

(3)

Our goal is to show that for any \(x_0\in\R\), we have \(\lim_{x\to x_0}a^x = a^{x_0}\).

We already have \(\lim_{n\to\infty}a^{\frac{1}{n}}=1\), and also \(\lim_{n\to\infty}a^{\frac{-1}{n}}=1\).

Which means that for any positive \(\epsilon\), there exists an \(N\in\N\), such that

\[ 1-\epsilon < a^{\frac{\pm1}{n}} < 1 + \epsilon \]

for all \(n\ge N\). It specially holds when \(n=N\).

Let \(\delta = \frac{1}{N}\). Then when \(|x-x_0| < \delta\), i.e., \(\frac{-1}{N} < x - x_0 < \frac{1}{N}\).

By monotonicity of exponentials, we have \(1-\epsilon < a^{\frac{-1}{N}} < a^{x-x_0} < a^{\frac{1}{N}} < 1 + \epsilon\).

Then

\[ \vert a^x - a^{x_0}| = a^{x_0} | a^{x-x_0} - 1 | < a^{x_0} \epsilon \]

What we need to do is choose the first positive number to be \(\frac{\epsilon}{a^{x_0}}\).

(4)

By (3), \(a^x\) is continuous, then \(\lim_{x\to0}a^x = a^0 = 1\).

Proposition 6.6 \(f(x)=a^x\) is a bijection if \(a>0\) and \(a\ne1\). The inverse map \(\log_a:(0,\infty)\to\R\) of exponential functions is also continuous.

Proof

(1)

(Injective) Proved by the monotonicity of exponentials

(Surjective)

For any real number \(r\), there must exist two \(x_1,x_2\in\R\) such that \(a^{x_1} < r < a^{x_2}\). It is easy to see the contradiction if no such kind of \(x_1,x_2\).

Then by intermediate value theorem, there exists a \(c\in(x_1,x_2)\), such that \(a^c=r\). And hence the map is surjective.

(2)

For \(x,x_0\in(0,\infty)\), let \(y=\log_ax, y_0=\log_ax_0\) which are both in \(\R\).

Considering \(|y-y_0| < \epsilon\) for any given \(\epsilon>0\), we have \(y < y_0+\epsilon\), with assuming \(a > 1\):

\[j a^y < a^{y_0+\epsilon}\\ a^y-a^{y_0} < a^{y_0}(a^\epsilon-1)\\ x-x_0 < \delta $$

for some \(\delta >0\).

Basically, for any \(\epsilon > 0\), there exists a \(\delta > 0\), such that if \(x - x_0 < \delta\) we have \(|y - y_0 = \log_a x - \log_a x_0| < \epsilon\), i.e. \(\lim_{x\to x_0} \log_a x = \log_a x_0\).

Proposition 6.7 Let \(a>0\), and for any \(b,c>0\).

  1. \(\log_a(bc) = \log_ab + \log_ac\)
  2. \(\log_ac = \log_ab \cdot \log_bc\) if \(b\ne1\)
  3. If \(a>1\), then \(\log_ab>\log_ac\), vice versa for $0

Proof (1)

\[ a^{\log_a(bc)}=a^{\log_ab+\log_ac}\\ bc=a^{\log_ab}\cdot a^{\log_ac}=bc \]

(2) and (3) are both similar with (1)

Proposition 6.8 \(\lim_{x\to\infty}\left(1+\frac{1}{x}\right)^x = \lim_{n\to\infty}\left(1+\frac{1}{n}\right)^n\) where \(x\in\R\) and \(n\in\N\).

And we call this limit as a number \(e\), Euler's number.

Proof We already know that \(\lim_{n\to\infty}\left(1+\frac{1}{n}\right)^n=e\), which means for any \(\forall \epsilon > 0\), there exists an \(N\in\N\), such that when the index \(n\) is greater than \(N\), we always have \(e - \epsilon < \left(1+\frac{1}{n}\right)^n < e + \epsilon\).

Let \(f(x) = \left(1+\frac{1}{x}\right)^x\), and \(F(n) = \left(1+\frac{1}{n}\right)^n\).

Firstly, we restrict \(x\) to be greater than \(n\) for some \(n\ge N\), i.e., \(n < x\).

We claim that \(F(n) < f(x)\).

Were this false, we have \(\left(1+\frac{1}{n}\right)^n \ge \left(1+\frac{1}{x}\right)^x\).

We choose \(x=2n\), then

\begin{align*} \left(1+\frac{1}{n}\right)^n $\ge \left(1+\frac{1}{x}\right)^x\\ &\ge \left(1+\frac{1}{2n}\right)^{2n}\\ \left(\frac{2n(n+1)}{n(2n+1)}\right)^n &\ge \left(1+\frac{1}{n}\right)^n \end{align*}

It is obvious that the numbers inside parenthesis are both greater than 1, then by monotonicity, we have

\begin{align*} \frac{2(n+1)}{2n+1} &\ge \frac{2n+1}{2n}\\ 0 &\ge 1 \end{align*}

This induces a contradiction. Therefore \(F(n) < f(x)\), then \(e - \epsilon < F(n) < f(x)\).

Secondly, by Archimedean Property, we can find a natural number \(m\), such that $x

We claim that \(f(x) < F(m)\).

Since \(m\) is even, there exists a \(k\in\N\) such that \(2k=m\). We can let \(m\) to be big enough so that \(x\) can become \(k\).

Were the claim false, we have

\begin{align*} \left(1+\frac{1}{x}\right)^x &\ge \left(1+\frac{1}{m}\right)^m \\ \left(1+\frac{1}{k}\right)^k &\ge \left(1+\frac{1}{2k}\right)^{2k} \end{align*}

Similar as before, we can obtain a contradiction if we keep on deducing.

Therefore, \(f(x) < F(m) < e + \epsilon\).

So with all that, for any \(\epsilon > 0\), there exist \(N\in\N\) and \(n,m\in\N\) with \(N < n < m\), such that when $n

\[ \lim_{x\to\infty}\left(1+\frac{1}{x}\right)^x = e = \lim_{n\to\infty}\left(1+\frac{1}{n}\right)^n \]

Proposition 6.9 \(\lim_{x\to0}\frac{a^x - 1}{x} = \log_e a =: lna\) if \(a>0\).

Proof Proof is divided into two parts.

(1) Consider the left limit.

Let \(t=a^x - 1\), then \(x = \log_a(1+x)\). Then

\begin{align*} \lim_{x\to0^+}\frac{a^x - 1}{x} &= \lim{t\to\infty}\frac{t}{\log_a(1+t)}\\ &=\lim{t\to\infty}\frac{1}{\log_a(1+t)^{\frac{1}{t}}}\\ &=\frac{1}{\log_ae} &=\ln a \end{align*}

(2) Consider the right limit.

\[ \lim_{x\to0^-}\frac{a^x - 1}{x} = \lim_{x\to0^-}a^x\frac{a^{-x} - 1}{-x} \]

Let \(t = -x\), then the above limit becomes \(\lim_{t\to0^+}a^{-t}\frac{a^t - 1}{t}\).

The former term is 0 as \(t\to0^+\) since \(a^x\) is continuous, and the later term is \(\ln e\) by (1).

Therefore, we have the left limit and right limit, with equal quantity \(\ln a\), then the limit is \(\ln a\).

Powered by Org Mode.